Last visit was: 28 Apr 2024, 13:44 It is currently 28 Apr 2024, 13:44

Close
GMAT Club Daily Prep
Thank you for using the timer - this advanced tool can estimate your performance and suggest more practice questions. We have subscribed you to Daily Prep Questions via email.

Customized
for You

we will pick new questions that match your level based on your Timer History

Track
Your Progress

every week, we’ll send you an estimated GMAT score based on your performance

Practice
Pays

we will pick new questions that match your level based on your Timer History
Not interested in getting valuable practice questions and articles delivered to your email? No problem, unsubscribe here.
Close
Request Expert Reply
Confirm Cancel
SORT BY:
Date
Math Expert
Joined: 02 Sep 2009
Posts: 92977
Own Kudos [?]: 619703 [7]
Given Kudos: 81613
Send PM
Math Expert
Joined: 02 Sep 2009
Posts: 92977
Own Kudos [?]: 619703 [0]
Given Kudos: 81613
Send PM
User avatar
Senior Manager
Senior Manager
Joined: 31 Mar 2016
Posts: 325
Own Kudos [?]: 195 [0]
Given Kudos: 197
Location: India
Concentration: Operations, Finance
GMAT 1: 670 Q48 V34
GPA: 3.8
WE:Operations (Commercial Banking)
Send PM
Intern
Intern
Joined: 23 Nov 2016
Posts: 44
Own Kudos [?]: 67 [1]
Given Kudos: 21
Location: United States (MN)
GMAT 1: 760 Q50 V42
GPA: 3.51
Send PM
Re: M16-28 [#permalink]
1
Bookmarks
Bunuel wrote:
Official Solution:

What is the area of a triangle enclosed by line \(2x+3y=6\), line \(y-x=2\) and \(X\)-axis on the coordinate plane?

A. 3
B. 4
C. 5
D. 8
E. 10


Look at the diagram below:



Lines \(y=-\frac{2}{3}x+2\) and \(y=x+2\) intersect at point \((0, 2)\). So the height of enclosed triangle is 2. Next, \(X\)-intercept of line \(y=-\frac{2}{3}x+2\) is \((3, 0)\) and \(X\)-intercept of line \(y=x+2\) is \((-2, 0)\), so the base of enclosed triangle is \(3-(-2)=5\). The area is \(\frac{1}{2}*base*height=\frac{1}{2}*5*2=5\).


Answer: C


If I were to set up a matrix with the three vertices of the triangle (-2,0), (0,2), (3,0), and divide the determinant of that matrix by 2, that would give the area of the triangle. However, I also believe that I could simply set up a 3x3 matrix with the equation of each line, and use that matrix to also find the area by dividing the absolute value of its determinant by 2. The latter is faster for this problem because it saves me from finding each vertex.

| 2 3 6 |
|-1 1 2 |
| 0 1 0 |

This gives a determinant of -10, and half of the absolute value of that is 5. Is using equations for each line actually a viable way to get the area, or is the vertex method the only guaranteed method? It works for this problem but I haven't been able to find it actually documented as a common method. My apologies if I missed this somewhere.
Intern
Intern
Joined: 15 Sep 2017
Posts: 19
Own Kudos [?]: 16 [0]
Given Kudos: 15
GMAT 1: 750 Q50 V42
GPA: 3.5
Send PM
Re: M16-28 [#permalink]
Great Question! Beautiful
Intern
Intern
Joined: 27 Apr 2016
Posts: 20
Own Kudos [?]: 3 [0]
Given Kudos: 4
Location: India
Schools: LBS (II)
GRE 1: Q156 V154
Send PM
Re: M16-28 [#permalink]
Bunuel wrote:
Official Solution:

What is the area of a triangle enclosed by line \(2x+3y=6\), line \(y-x=2\) and \(X\)-axis on the coordinate plane?

A. 3
B. 4
C. 5
D. 8
E. 10


Look at the diagram below:



Lines \(y=-\frac{2}{3}x+2\) and \(y=x+2\) intersect at point \((0, 2)\). So the height of enclosed triangle is 2. Next, \(X\)-intercept of line \(y=-\frac{2}{3}x+2\) is \((3, 0)\) and \(X\)-intercept of line \(y=x+2\) is \((-2, 0)\),so the base of enclosed triangle is \(3-(-2)=5\). The area is \(\frac{1}{2}*base*height=\frac{1}{2}*5*2=5\).


Answer: C



Hi, Did not understand the highlighted part. Derived the height but didn't get the base calc. Please help! How is X coordinate 3 and -2?

TIA
Math Expert
Joined: 02 Sep 2009
Posts: 92977
Own Kudos [?]: 619703 [1]
Given Kudos: 81613
Send PM
Re: M16-28 [#permalink]
1
Kudos
Expert Reply
archimaitreya25 wrote:
Bunuel wrote:
Official Solution:

What is the area of a triangle enclosed by line \(2x+3y=6\), line \(y-x=2\) and \(X\)-axis on the coordinate plane?

A. 3
B. 4
C. 5
D. 8
E. 10


Look at the diagram below:



Lines \(y=-\frac{2}{3}x+2\) and \(y=x+2\) intersect at point \((0, 2)\). So the height of enclosed triangle is 2. Next, \(X\)-intercept of line \(y=-\frac{2}{3}x+2\) is \((3, 0)\) and \(X\)-intercept of line \(y=x+2\) is \((-2, 0)\),so the base of enclosed triangle is \(3-(-2)=5\). The area is \(\frac{1}{2}*base*height=\frac{1}{2}*5*2=5\).


Answer: C



Hi, Did not understand the highlighted part. Derived the height but didn't get the base calc. Please help! How is X coordinate 3 and -2?

TIA


The x-intercept of a line, (x, 0), is a point at which the line crosses the x-axis.

To get x-intercept of line \(2x+3y=6\), put y = 0 --> x = 3. So, the x-intercept of \(2x+3y=6\) is (3, 0).
To get x-intercept of line \(y-x=2\), put y = 0 --> x = -3. So, the x-intercept of \(y-x=2\) is (-2, 0).

The distance between, (3, 0) ans (-2, 0) = is the length of a base = 5.
Intern
Intern
Joined: 27 Apr 2016
Posts: 20
Own Kudos [?]: 3 [0]
Given Kudos: 4
Location: India
Schools: LBS (II)
GRE 1: Q156 V154
Send PM
Re: M16-28 [#permalink]
Bunuel wrote:
archimaitreya25 wrote:
Bunuel wrote:
Official Solution:

What is the area of a triangle enclosed by line \(2x+3y=6\), line \(y-x=2\) and \(X\)-axis on the coordinate plane?

A. 3
B. 4
C. 5
D. 8
E. 10


Look at the diagram below:



Lines \(y=-\frac{2}{3}x+2\) and \(y=x+2\) intersect at point \((0, 2)\). So the height of enclosed triangle is 2. Next, \(X\)-intercept of line \(y=-\frac{2}{3}x+2\) is \((3, 0)\) and \(X\)-intercept of line \(y=x+2\) is \((-2, 0)\),so the base of enclosed triangle is \(3-(-2)=5\). The area is \(\frac{1}{2}*base*height=\frac{1}{2}*5*2=5\).


Answer: C



Hi, Did not understand the highlighted part. Derived the height but didn't get the base calc. Please help! How is X coordinate 3 and -2?

TIA


The x-intercept of a line, (x, 0), is a point at which the line crosses the x-axis.

To get x-intercept of line \(2x+3y=6\), put y = 0 --> x = 3. So, the x-intercept of \(2x+3y=6\) is (3, 0).
To get x-intercept of line \(y-x=2\), put y = 0 --> x = -3. So, the x-intercept of \(y-x=2\) is (-2, 0).

The distance between, (3, 0) ans (-2, 0) = is the length of a base = 5.



Thanks! Got it!
Math Expert
Joined: 02 Sep 2009
Posts: 92977
Own Kudos [?]: 619703 [1]
Given Kudos: 81613
Send PM
Re: M16-28 [#permalink]
1
Bookmarks
Expert Reply
Similar questions to practice:



Hope it helps.
Math Expert
Joined: 02 Sep 2009
Posts: 92977
Own Kudos [?]: 619703 [0]
Given Kudos: 81613
Send PM
Re: M16-28 [#permalink]
Expert Reply
I have edited the question and the solution by adding more details to enhance its clarity. I hope it is now easier to understand.
GMAT Club Bot
Re: M16-28 [#permalink]
Moderator:
Math Expert
92977 posts

Powered by phpBB © phpBB Group | Emoji artwork provided by EmojiOne